What is Equilibrium: Definition and 1000 Discussions

Thermodynamic equilibrium is an axiomatic concept of thermodynamics. It is an internal state of a single thermodynamic system, or a relation between several thermodynamic systems connected by more or less permeable or impermeable walls. In thermodynamic equilibrium there are no net macroscopic flows of matter or of energy, either within a system or between systems.
In a system that is in its own state of internal thermodynamic equilibrium, no macroscopic change occurs.
Systems in mutual thermodynamic equilibrium are simultaneously in mutual thermal, mechanical, chemical, and radiative equilibria. Systems can be in one kind of mutual equilibrium, though not in others. In thermodynamic equilibrium, all kinds of equilibrium hold at once and indefinitely, until disturbed by a thermodynamic operation. In a macroscopic equilibrium, perfectly or almost perfectly balanced microscopic exchanges occur; this is the physical explanation of the notion of macroscopic equilibrium.
A thermodynamic system in a state of internal thermodynamic equilibrium has a spatially uniform temperature. Its intensive properties, other than temperature, may be driven to spatial inhomogeneity by an unchanging long-range force field imposed on it by its surroundings.
In systems that are at a state of non-equilibrium there are, by contrast, net flows of matter or energy. If such changes can be triggered to occur in a system in which they are not already occurring, the system is said to be in a meta-stable equilibrium.
Though not a widely named "law," it is an axiom of thermodynamics that there exist states of thermodynamic equilibrium. The second law of thermodynamics states that when a body of material starts from an equilibrium state, in which, portions of it are held at different states by more or less permeable or impermeable partitions, and a thermodynamic operation removes or makes the partitions more permeable and it is isolated, then it spontaneously reaches its own, new state of internal thermodynamic equilibrium, and this is accompanied by an increase in the sum of the entropies of the portions.

View More On Wikipedia.org
  1. Ander99

    Engineering Calculating Equilibrium Points in OVA-Opamp Circuit with Ideal Diodes

    Hi everyone, I need help with this exercise becouse I don't know how to calculate the equilibrium points of a circuit. The original exersice is the following: Assume that the open loop dynamic response of the OVA-opamp gets captured by Fig. 1(a) and consider the circuit of Fig. 1(b). Consider...
  2. Justin_Lahey

    Particle in equilibrium (balancing forces on an object on an incline)

    Hi, I’m wondering if someone can help me understand this question. I can find a resultant force/vector when given an initial angle but I’m stuck here when the only information is the two magnitudes. I think I’m solving for the unknowns but a little lost on how or what equation I should be using...
  3. Butterfly41398

    Equilibrium of coplanar forces

    The answer is 280N but I don't have any idea hoe to get to the anseer. Atleast a hint would help.
  4. Butterfly41398

    Equilibrium of concurrent forces -- 3 Pulleys and 2 Weights

    Help, this one is completely different, the weird pulley is making it hard for me and I don't even know what to do with it and where to start. The answer is 185N but I want to know how.
  5. Y

    Equilibrium for a rotation lab

    So for the first one on the left is it T= 2g*20=40Nm ? Our professor said it is clockwise I just don't understand how she gets that conclusion.
  6. lakuzamalakuzam

    Problem on Equilibrium ( STATICS )

    so i was wondering what assumptions needed to make this solvable and ' support at both thrust collars can totally support the moments of 400 Nm before they slip' means which axis of moment
  7. C

    Help finding the equilibrium position of an electric field

    I seem completely lost at this. I barely know where to begin. I know that the forces will sum to 0 but the vectoral nature of the question is really confusing me. Best I have is that the distance between e and q2 has to be sqrt(2) times the distance between e and q1. I don't know where to go...
  8. I

    Calculate Equilibrium Constant for Polyprotic Acid Reaction

    The principal equilibrium in a solution of NaHCO3 is HCO3-(aq) + HCO3-(aq) <-> H2CO3(aq) + CO32-(aq) Calculate the value of the equilibrium constant for this reaction. My solution: This overall reaction is the same as the sum of the following reactions: HCO3-(aq) <-> H2CO3(aq) + OH-(aq)...
  9. goodOrBad

    Block A Reaches Equilibrium: 92N Force

    I tried to solve it via equilibrium equations and I got 92N for block A
  10. LCSphysicist

    Interesting exercise: Torque, radiation, momentum and equilibrium

    t is Torque I is the inertia moment P is the power c is the constant light speed r is the spot distance to the fiber p is the torsional constant theta is what we want In the equilibrium $$t = 0$$ $$ F\Delta T = \frac{E}{c} = \frac{P\Delta T}{c} => *F* = \frac{P}{c} (1) $$ This will be the...
  11. mcrooster

    Help finding the vertical component of the reaction at C

    Hello everyone, For this equation, I have found Ay as 0.677kN by using Ma = 1.5kN(7m)-Ay(15.5m). To find Cy, would I be using the sum of forces to find the answer? For example Fy = Ay - F2 - F1sin55 + Cy = 0 This would then get Cy answer as 2.05kN
  12. mcrooster

    How to find Tb and Moments 3D equilibrium of rigid bodies

    The boom is supported by a ball-and-socket joint at AA and a guy wire at BB Hey guys, I am stuck with this question in find the Tension in B and the moments around A. I have done plenty of 2d Tension questions but not a 3D one.
  13. Traced

    Chemistry Equilibrium K Values: Problem Solving Tips

    I know what K does, what can change K, and I know about the equilibrium equation. I don't know how to use that to answer this problem. I'm not given any chemical reactions or their K values. How should I start this problem?
  14. I

    Activity - Equilibrium Expressions

    When I use the concept of activity to express the equilibrium constant expression, for either equilibrium pressures or concentrations, the units cancel and the value of K has no units (which is how K is customarily reported). But because of the difference in reference states (the reference...
  15. S

    Chemistry Can anyone check my answers for these equilibrium problems?

    The following are my works for the problems: I put squares on my final answers! Thank you!
  16. il postino

    Chemistry Calculate DeltaH between 700K and 800K

    Hi all Knowing that ##kp## is: ##k_p=\frac{pO_2.(pNO)^2}{(pNO_2)^2}## And knowing these relationships between the partial pressures, I obtained: At 700 K: ##pNO = 0.872.pNO2## ##k_p=pO_2.(0.872)^2## At 800 K: ##pNO = 2.5.pNO2## ##k_p=pO_2.(2.5)^2## Furthermore it is known that ##pNO + pNO +...
  17. D

    Phase Equilibrium of Liquid and Vapor Under External Pressure

    Suppose you have a container of water at a given temperature T (say normal room temperature) with a vacuum above it. Presumably water will evaporate until there is sufficient vapor that the pressure of it above the water is the SVP for that temperature. Now suppose that there is air above the...
  18. M

    Engineering Solve Value of R: Moment & Equilibrium

    Summary:: I need some help with moment when using equations of equilibrium Edit: Sorry I forgot to say what the question was! The question was to solve the value of R based on the diagram below alone. I was solving the question below and I tried to resolve R into a force that is perpendicular...
  19. S

    Why Does Torque Equilibrium Not Solve for Fcy in Static Equilibrium Problems?

    I have two questions regarding this problem 1. The following is my attempt to solve Fcy using the fact that the addition of torque = 0. I considered force mg and Fcy and their lever arms to set up the equilibrium. But this method seems not to work? Why? 2.At the bottom of the question, it...
  20. halleff

    Diffusion current and carrier concentration equilibrium (unbiased)

    Suppose you have a non-uniformly doped piece of semiconductor (without an applied bias) such that the acceptor dopant concentration Na(x) decreases from left to right (as x increases). In this case, the equilibrium hole distribution p(x) will not be uniform since then there would be a net drift...
  21. T

    Exploring the Grand Partition Function for an Einstein Solid

    $$Q_{(\alpha, \beta)} = \sum_{N=0}^{\infty} e^{\alpha N} Z_{N}(\alpha, \beta) \hspace{1cm} (3.127)$$ Where ##Q## is the grand partition function, ##Z_N## is the canonical partition function and: $$\beta = \frac{1}{kT} \hspace{1cm} \alpha = \frac{\mu}{kT} \hspace{1cm} (3.128)$$ In the case of an...
  22. LCSphysicist

    Measuring Blocks Equilibrium Displacements

    "The displacements of the blocks from equilibrium are both measured to the right. Block 1 has a mass of 15 grams and block 2 a mass of 10 grams. The spring constants of the springs are shown in dynes/cm." I don't know if i understood very well the notation, but i interpreted as F(t) acting only...
  23. il postino

    Chemistry Chemistry equilibrium - Remove H2

    It is desired to remove the molecular hydrogen present in a flask. What will be better to introduce, ##Cl_2## or ##Br_2##? Why? ##2 HCl <-> H_2 + Cl_2## (1) ##k = 3,2.10^{-14}## ##H_2 + Br_2 <->2 HBr## (2) ##k = 2.10^9## I thought about turning equation (1) to make the...
  24. D

    Conceptual Problem - Static Equilibrium

    I am not sure whether I can prove like this, I am trying to explain it with the angle .
  25. R

    Trouble with Static Equilibrium

    My initial response to seeing the figure is what is ##F_{ua}## and where does it come from? How was i supposed to know it was there is they didn't give a picture? So for part a, ##F_{ua}## doesn't play a role because it exerts no torque, and apparently we are supposed to use the torque equation...
  26. T

    Small deviations from equilibrium and Lagrange multipliers

    According to the book "Principles of Statistical Mechanics" by Amnon Katz, page 123, ##\alpha## must be such that ##\exp ( -\alpha N ) ## can be expanded in powers of ##\alpha## with only the first order term kept. Is this the necessary and sufficient condition for small deviations from...
  27. T

    Equilibrium Temp of two fluids

    Not really sure how to start this one.
  28. S

    Think of a body which is at rest but not in equilibrium

    When a ball is thrown upward it becomes at rest at maximum height, at this it is not in equilibrium although it is at rest. It is not at equilibrium because force of gravity is acting on it? Still I cannot find good explanation from exam point of view.I also cannot find the figure/diagram.
  29. Dman0500

    Dynamic Equilibrium -- Acceleration of a rock thrown from a bridge

    I know the acceleration of the rock is equal to g, but why. If we neglect air resistance, what is actually making the rock fall? Wouldn't it be that g overcomes the acceleration of the y plane at some point so the rock starts coming down or in this case accelerate more by throwing below 0 degrees?
  30. K

    What does it mean that a rigid body is in equilibrium?

    So I have always been thinking that equilibrium means that an object is not moving or having constant acceleration. On a webside they said: " A rigid body is in equilibrium when it is not undergoing a change in rotational or translational motion. " To me it sounds like the object then must not...
  31. K

    Help with understanding this rigid-body equilibrium problem

    So here are two problems I have been working with lately: I have solved both, so I don't need the help to find the answers. The thing that confusses me is which object one should choose to apply the equations above. When it comes to the first problem, we apply the equations to the pole, but...
  32. K

    Understanding rigid-body equilibrium problem

    So I have this problem The soultion to the problem gives me this drawing But I don't understand what ##F_h## and ##F_v## is
  33. J

    Static equilibrium: mass on beam

    A diagram of the physical situation is below: Choosing the positive ##y## direction to be upwards and the positive direction of rotation to be counterclockwise, Newton's linear second law gives: $$-m_D g + F_L + F_R = 0$$ where ##F_L## is the magnitude of the force exerted on the bar by the...
  34. S

    Equilibrium value for carbonation level in beer

    Abs Pressure: Gauge press: 1 bar G = 100,000 Pa = 0.987 atm 0.1 bar G = 0.0987 atm Atm press = 101.3 kPa = 1 atm Hydrostatic press: average CO2 occurs 2.5m up the vessel (9.81 ms-2)(1010kgm-3)(5/2) = 24,770.25 Pa 1 Pa = 9.869x10^-6 atm 24,770.25 Pa = 0.2445 atm Abs press= 0.0987 atm + 1 atm...
  35. archaic

    Static equilibrium problem: Finding tension

    The diagonal component of the boom's weight is ##R=w\cos\frac{\pi}{6}=\frac{\sqrt 3}{2}w##, and, considering ##R## as a "reaction", we have ##R_x=R\cos\frac{\pi}{3}=\frac{\sqrt 3}{4}w## and ##R_y=R\sin\frac{\pi}{3}=\frac 34w##. I will also have...
  36. archaic

    Static equilibrium: Placing a fulcrum

    I don't understand the question; they're telling me that the fulcrum is just under the center of gravity of the bar, but that I also need to find its position. In any case, I suppose that the fulcrum's position is ##x##. Let the weight at the left end be ##w_1##, at the right end be ##w_2##...
  37. archaic

    Static equilibrium and a diving board

    I take the left end as reference for torques: $$\sum\tau=F_\text{support}+\frac 32\times280+3\times500=0\Leftrightarrow F_\text{support}=-192.\times10^1\,N$$ It's facing up. I take the right end as reference for torques: $$\sum\tau=-192.\times10^1\times2+\frac 32\times280+3F=0\Leftrightarrow...
  38. E

    Does pressure affect equilibrium vapor pressure (or RH)?

    I'm wondering if I'm on the right track and if anyone is willing to steer me on if not: Equilibrium vapor pressure (EVP—also referred to as saturation vapor pressure) is dependent only on temperature. Outside pressure has no bearing. Now, of course, with lower external pressure (atmospheric)...
  39. gaurav_samanta

    Can heat transfer break an equilibrium?

    Here is one scenario: I have placed a metal in my room which is at room temperature. Air has little much heat capacity and metals don't like to store heat. Would heat transfer occur? Whatif I set the temperature of both to a certain degree where it crosses heat capacity of metal but not of air?
  40. S

    Static Equilibrium Problem (concept question)

    (figure)I was able to solve the problem and get the right answer. However, I solved it more with intuition, and I do not get how the force of the person pulling down on the rope makes the force to the left of the backpack equal to force to the right of the backpack.
  41. il postino

    Chemistry Equilibrium temperature of a water and ice system

    Equal masses of ice at –10ºC and water at 80ºC are placed in an insulated container and allowed to reach thermal equilibrium. Calculate the equilibrium temperature Data: Water(ice): 37,65 J/mol.K Agua (l): 75,29 J/mol.K ## Lf = 6011 J/mol ## I solved it this way: ## -Q_{l} = Q_{ice} ## ##...
  42. Eggue

    Missing factor of 2 in an equilibrium problem

    I honestly have no idea where I'm going wrong i've checked my differentiation using a derivative calculator but the answer is the same. The answer from the book is \frac{L}{2} + \frac{h^2}{L} Working:
  43. cs44167

    Why Is My Calculation of Electrostatic Equilibrium Incorrect?

    I set the electrostatic force exerted by the object at (0,0) and (3,0) equal to each other, dividing out k and q2. I was left with q1/d^2 for both terms and substituted in the given charges for each object. I then replaced d^2 for the object at (0,0) with “x^2” and d^2 for the object at (3,0)...
  44. Adesh

    Three doubts in a paragraph on the equilibrium of Incompressible fluids

    These are the images from Sommerfeld’s Lectures on Theoretical Physics, Vol 2 chapter 2, section 6, Equilibrium of Incompressible Fluids. Image 1 Image 2 Doubt 1 : What does it mean for a force to act on a fluid volume? Force acts on a point, force may act on a surface but I’m unable to...
  45. S

    Equilibrium concentration of co2 in beer

    Absolute pressure is gauge pressure + atmos press Guage press = 1.29 bar G 1 bar = 10^5 Pa 1.29 bar = 129,000 Pa Atmos Pres = 1.01x10^5 Pa Absolute pressure = 230,000 Pa Henrys law: P=KhC 230000Pa = (116x10^3Pa)C 1.98 mole fraction =C 70% CO2 = 1.387 mole fraction 44g Co2 in 1 mole 61.07g...
  46. tanaygupta2000

    Statistical Mechanics: Two systems reaching an equilibrium temperature

    First I found partition functions of both the systems and hence total energies of them using above formulas. Z(A) = (1 - e-ε/kT)-1 and Z(B) = (1 + e-ε/kT) Then I equated these values to the given values of total energies. I got: For System A, T(A) = ε/kln(2) > 0 For System B, T(B) =...
  47. S

    Calculate the equilibrium concentration of CO2 in beer in grams/liter

    Absolute pressure = Gauge press + Atmos press Atmos press = 105 Pa Co2 press = 0.1 bar g = 10000Pa Abs press = 10,105 Pa Hydrostatic pressure = absolute press + (density)(grav)(height at midpoint) = 10,105 + (1008)(9.81)(10) 10,8989.8 Pa Calc CO2 conc by henrys law P=KHC C=P/KH =...
Back
Top